LSAT and Law School Admissions Forum

Get expert LSAT preparation and law school admissions advice from PowerScore Test Preparation.

 Harry0829
  • Posts: 7
  • Joined: Jan 17, 2019
|
#61893
Hi,

On page 305, Question 2, answerchoice (C),

Your explanation is "Some individuals taking antidepressants that cause weight gain will gain weight"
That is true, but I feel like this is not what the answerchoice indicates, which is basically "Some patients taking antidepressant drugs gain weight", and there seems to be no guarantee against the claim that patients who take antidepressant drugs will all choose antidepressant drugs that are not going to cause weight gain (a claim that is fair because the stimulus states "most" antidepressant drugs cause weight gain). So assuming that there are thirty patients who take antidepressant drugs, and 100 kinds of antideressant drugs, 51 of which cause weight gain, what if all thirty patients taking antidepressant drugs choose among 49? (Admittedly this scenario is unrealistic, but I think logic in LSAT must be bulletproof against all such scenarios)
In this scenario, I don't think you can equate "some patients taking antidepressant drugs" (in answer choice (C)) with "some individuals taking antidepressants that cause weight gain" (in your explanation).

So, my question is, is the stimulus assuming that Patients who take antidepressant drugs :dbl: antidepressant drugs?
Which is to say that all patients who take antidepressant drugs take antidepressant drugs, as well as all antidepressant drugs are taken by patients who take antidepressant drugs? I feel like only under such circumstance would this answer choice be correct, namely Patients who take antidepressant drugs :dbl: antidepressant drugs :most: gain weight which indicates Patients who take antidepressant drugs :some: gain weight,
without this assumption, Patients who take antidepressant drugs :arrow: antidepressant drugs :most: gain weight, which does not necessarily mean this answer choice is correct.
 Adam Tyson
PowerScore Staff
  • PowerScore Staff
  • Posts: 5153
  • Joined: Apr 14, 2011
|
#61967
You've hit on a good point here, Harry - even though most antidepressants cause weight gain, there is no guarantee than anyone ever takes those drugs. However, you have to temper that analysis with two concepts:

1. The question stem is a "most strongly supported", not a pure "must be true". That means that the correct answer will be a reasonable inference to make, supported by the stimulus, but not absolutely necessary. Answer C is a reasonable inference to make - someone is probably taking at least one of those drugs that cause weight gain, since those are the majority of all antidepressants. The stimulus supports this answer, if it doesn't absolutely prove it.

2. You are supposed to pick the best answer of the five presented, rather than argue with that answer because it is imperfect. There is no other answer here that gets any support from the stimulus, while the correct answer at least gets some support and seems on its face to be reasonable, subject only to a very limited exceptional hypothetical.

The logic of the questions and answers on the LSAT is NOT always bulletproof, but the right answer IS always right for a good reason.

I didn't understand this question you asked, or your analysis after that point:
So, my question is, is the stimulus assuming that Patients who take antidepressant drugs :dbl: antidepressant drugs?
But I can tell you that the answer assumes that at least one person takes one of the antidepressants that causes weight gain, and that is all you need to pick this answer. The only way it would be incorrect is if no patients take any of those particular drugs, and that seems like a stretch. Possible, sure, but we aren't looking to shoot holes in answer, but only to pick the answer that is better than the other four.
User avatar
 Dave Killoran
PowerScore Staff
  • PowerScore Staff
  • Posts: 5853
  • Joined: Mar 25, 2011
|
#61979
Harry0829 wrote: there seems to be no guarantee against the claim that patients who take antidepressant drugs will all choose antidepressant drugs that are not going to cause weight gain (a claim that is fair because the stimulus states "most" antidepressant drugs cause weight gain)
Hi Harry,

You raise an interesting discussion point, and I want to address the heart of the issue which I quoted above. While what you say sounds true, this is not how LSAC would view this, and they would cite "commonsense standards" as the reason as well as the way they presented the information (as a declarative statement). So let's take a look at that!

Every LR sections is preceded by the following directions (with italics added):

  • Directions: Each question in this section is based on the reasoning presented in a brief passage. In answering the questions, you should not make assumptions that are by commonsense standards implausible, superfluous, or incompatible with the passage. For some questions, more than one of the choices could conceivably answer the question. However, you are to choose the best answer; that is, choose the response that most accurately and completely answers the question and mark that response on your answer sheet.
Now, that phrase may look rather innocent (or inscrutable, depending on your take), but what it allows LSAC to do is say that certain element within questions fall under the "this is the way things typically work in the real world" defense for any given question. There are millions of examples one could cite, but here are a few:

  • Supply and demand generally works so when demand rises, prices often do as well.

    Humans require air to live.

    One goal of a police force is to reduce crime.

    National governments often provide defense for the country.

    Cigarettes are unhealthy.
Now, the application of that idea to this problem comes from the sentence that states, "Most antidepressant drugs cause weight gain." In this case, LSAC will say that the population of people taking antidepressants is significant enough that some will indeed take those that cause weight gain (the presumption then, that people exist within that category). There's further justification for that belief in that the basic workings of the pharma industry would be thought to be known, and drug companies would not offer a majority of products in a category unless those products had users. In other words, LSAC would say this is a situation where you know people use those particular weigh-gain causing drugs, and to think otherwise is counter (or implausible) to what everyone expects about drugs like that.

Now, if the above is the case, where is the guide that tells you what you are supposed to accept and not accept? That's where it gets tricky—there isn't one. The guide is whether it's a situation or scenario the overwhelming majority of college-educated people would be expect to know this is how things work (as with the above examples, and with how drugs/economics works). Tough to accept at times, but an excellent point to ponder as you look at other LR questions.

Please let me know if that helps. Thanks!
 Adam Tyson
PowerScore Staff
  • PowerScore Staff
  • Posts: 5153
  • Joined: Apr 14, 2011
|
#61982
Dave adds a good point here, and we are back to the section directions again! Pick the best answer and don't spin out elaborate hypothetical situations to argue against an answer choice.

Also, consider this: the text says that those drugs cause weight gain. Not that they could cause or would cause weight gain - they do. In order for that to be true, there must be at least one instance where they do, in fact, do that, and that means someone took one of them.
 Harry0829
  • Posts: 7
  • Joined: Jan 17, 2019
|
#62034
I agree with you that the assumption I made to refute answer choice (C) was definitely not commonsensical.
Thank you for your help and I will keep the directions in mind at all times!

Get the most out of your LSAT Prep Plus subscription.

Analyze and track your performance with our Testing and Analytics Package.